LSAT and Law School Admissions Forum

Get expert LSAT preparation and law school admissions advice from PowerScore Test Preparation.

 Administrator
PowerScore Staff
  • PowerScore Staff
  • Posts: 8917
  • Joined: Feb 02, 2011
|
#55809
Complete Question Explanation
(The complete setup for this game can be found here: lsat/viewtopic.php?t=15077)

The correct answer choice is (B)

The conditions in the question stem establish a horizontal ZT block:

pt43_j04_g3_q17a.png

Because we have already established that Z can only be sold to the museum as either the second or third period piece, the block must be placed so that Z is from the second period and T is from the third period:

pt43_j04_g3_q17b.png

At this point, we can apply the first rule and third rules, which serve to establish that S is from the first period and that V is sold to the private collector:
pt43_j04_g3_q17c.png
The only remaining variables yet to be placed are Q and R. From the second rule we know that
pt43_j04_g3_5.png
and thus Q must be from the third period (otherwise it would violate the second rule). Consequently, Q must be sold to the private collector, and R, the random, occupies the museum’s first period:

pt43_j04_g3_q17d.png

Using this analysis, the problem is easy.

Answer choices (A), (C), (D), and (E): Each of these answer choices cannot occur, and therefore each is incorrect.

Answer choice (B): This is the correct answer choice.
You do not have the required permissions to view the files attached to this post.

Get the most out of your LSAT Prep Plus subscription.

Analyze and track your performance with our Testing and Analytics Package.